Endocrine (Thyroid) Questions

¡Supera tus tareas y exámenes ahora con Quizwiz!

A client is being tested for Graves' disease. The nurse knows that which one of the following lab panels would be used to rule out this condition? Liver panel Cardiac panel Renal panel Thyroid panel

Thyroid panel Graves' disease is an autoimmune disease associated with hyperthyroidism. The client would need a thyroid panel in order to rule out the condition.

6. Which of the following are not a treatment for Thyroid Storm? A. Propylthiouracil (PTU) B. Synthroid C. Inderal D. Glucocorticoids

Synthroid is a medication treatment for HYPOthyroidism. All the other options are for HYPERthyroidism.

Which range best describes normal levels of thyroid-stimulating hormone? 12.2 to 15 mU/L 5 to 8.9 mU/L 35 to 45 mU/L 0.4 to 4.0 mU/L

0.4 to 4.0 mU/L Thyroid-stimulating hormone (TSH) is a test of thyroid function that can determine if a client has hypo- or hyperthyroidism. The normal range of values for thyroid stimulating hormone is 0.4 to 4.0 mU/L. If the number is low, it can indicate hyperthyroidism or secondary hypothyroidism. If the number is high, it can indicate primary hypothyroidism.

8. Which of the following medication orders should a nurse question if ordered on a patient with thyroid storm? A. Propylthiouracil "PTU" for a 25 year old who is 8 weeks pregnant B. Aspirin as needed for a fever greater than 102.2 'F C. Inderal for a patient who reports having insomnia D. Tapazole for a 30 year old having complaints of a headache

A patient who has hyperthyroidism or thyroid storm should NEVER take salicylate (ex: aspirin) because it canincrease thyroid hormones. All the other options are correct or insignificant for why the patient is taking the medication.

A client visits the primary health care provider's office for a routine physical examination and reports a new onset of intolerance to cold. Since hypothyroidism is suspected, which additional information would be noted during the client's assessment? Weight loss and tachycardia Complaints of weakness and lethargy Diaphoresis and increased hair growth Increased heart rate and respiratory rate

Complaints of weakness and lethargy Rationale: Weakness and lethargy are the most common complaints associated with hypothyroidism. Other common symptoms include intolerance to cold, weight gain, bradycardia, decreased respiratory rate, dry skin, and hair loss.

The nurse is performing an assessment on a client with a diagnosis of myxedema (hypothyroidism). Which assessment finding should the nurse expect to note in this client? Dry skin Thin, silky hair Bulging eyeballs Fine muscle tremors

Dry skin Rationale: Myxedema is a deficiency of thyroid hormone. The client will present with a puffy, edematous face, especially around the eyes (periorbital edema), along with coarse facial features; dry skin; and dry, coarse hair and eyebrows. The remaining options are noted in the client with hyperthyroidism.

A client with suspected primary hyperparathyroidism is undergoing diagnostic testing. The nurse would assess for which as a manifestation of this disorder? Polyuria Diarrhea Polyphagia Weight gain

Polyuria Rationale: Hypercalcemia classically occurs with hyperparathyroidism. Elevated serum calcium levels produce osmotic diuresis, making polyuria the correct option. The other manifestations listed are not associated with this disorder.

The nurse is taking a health history for a client with hyperparathyroidism. Which question would elicit information about this client's condition? "Do you have tremors in your hands?" "Are you experiencing pain in your joints?" "Do you notice swelling in your legs at night?" "Have you had problems with diarrhea lately?"

"Are you experiencing pain in your joints?" Rationale: Hyperparathyroidism is associated with oversecretion of parathyroid hormone (PTH), which causes excessive osteoblast growth and activity within the bones. When bone reabsorption is increased, calcium is released from the bones into the blood, causing hypercalcemia. The bones suffer demineralization as a result of calcium loss, leading to bone and joint pain and, sometimes, pathological fractures. Tremors and diarrhea relate to assessment findings of hypoparathyroidism. Swelling in the legs at night is unrelated to hyperparathyroidism.

The nurse has provided instructions to the client with hyperparathyroidism regarding home care measures to manage the symptoms of the disease. Which statement by the client indicates a need for further instruction? "I should avoid bed rest." "I need to avoid doing any exercise at all." "I need to space activity throughout the day." "I should gauge my activity level by my energy level."

"I need to avoid doing any exercise at all." Rationale: The client with hyperparathyroidism should pace activities throughout the day and plan for periods of uninterrupted rest. The client should plan for at least 30 minutes of walking each day to support calcium movement into the bones. The client should be instructed to avoid bed rest and use energy levels as a guide to activity. The client also should be instructed to avoid high-impact activity or contact sports.

The home care nurse visits a client with a diagnosis of hyperparathyroidism who is taking furosemide and provides dietary instructions to the client. Which statement by the client indicates a need for additional instruction? "I need to eat foods high in potassium." "I need to drink at least 2 to 3 L of fluid daily." "I need to eat small, frequent meals and snacks if nauseated." "I need to increase my intake of dietary items that are high in calcium."

"I need to increase my intake of dietary items that are high in calcium." Rationale: The aim of treatment in the client with hyperparathyroidism is to increase the renal excretion of calcium and decrease gastrointestinal absorption and bone resorption of calcium. Dietary restriction of calcium may be used as a component of therapy. The client should eat foods high in potassium, especially if the client is taking furosemide. Drinking 2 to 3 L of fluid daily and eating small, frequent meals and snacks if nauseated are appropriate instructions for the client.

13. What characteristic is related to Hashimoto's thyroiditis? a. Enlarged thyroid gland b. Viral-induced hyperthyroidism c. Bacterial or fungal infection of thyroid gland d. Chronic autoimmune thyroiditis with antibody destruction of thyroid tissue

13. d. In Hashimoto's thyroiditis, thyroid tissue is destroyed by autoimmune antibodies. An enlarged thyroid gland is a goiter. Viral-induced hyperthyroidism is subacute granulomatous thyroiditis. Acute thyroiditis is caused by bacterial or fungal infection

8. A patient who is in her first trimester of pregnancy is diagnosed with hyperthyroidism. Which medication do you suspect the patient will be started on? A. Propylthiouracil (PTU) B. Radioactive Iodine C. Tapazole D. Synthroid

The answer is A: Propylthiouracil (PTU) is the only anti-thyroid medication that can be used during the 1st trimester of pregnancy.

23. Patient-Centered Care: The nurse has identified the nursing diagnosis of fatigue for a patient who is hypothyroid. What should the nurse do while caring for this patient? a. Monitor for changes in orientation, cognition, and behavior. b. Monitor for vital signs and cardiac rhythm response to activity. c. Monitor bowel movement frequency, consistency, shape, volume, and color. d. Assist in developing well-balanced meal plans consistent with energy expenditure level

23. b. Cardiorespiratory response to activity is important to monitor in this patient to determine the effect of activities and plan activity increases. Monitoring changes in orientation, cognition, and behavior are interventions for impaired memory. Monitoring bowels is needed to plan care for the patient with constipation. Assisting with meal planning will help the patient to lose weight if needed

24. Priority Decision: When replacement therapy is started for a patient with long-standing hypothyroidism, what is most important for the nurse to monitor for in the patient? a. Insomnia c. Nervousness b. Weight loss d. Dysrhythmias

24. d. All these manifestations may occur with treatment of hypothyroidism. However, as a result of the effects of hypothyroidism on the cardiovascular system, when thyroid replacement therapy is started, myocardial oxygen consumption is increased and the resultant oxygen demand may cause angina, cardiac dysrhythmias, and heart failure, so monitoring for dysrhythmias is most importa

13. A patient is 6 hours post-opt from a thyroidectomy. The surgical site is clean, dry and intact with no excessive swelling noted. What position is best for this patient to be in? A. Fowler's B. Prone C. Trendelenburg D. Semi-Fowler's

The answer is D: Semi-Fowler's

2. A patient is admitted with thyroid storm. Which sign and symptoms are NOT present with this condition-SELECT ALL THAT APPLY? A. Temperature of 104.9'F B. Heart rate of 125 bpm C. Respirations of 42 D. Heart rate of 20 bpm E. Intolerance to cold F. Restless

Bradycardia (heart rate of 20 bpm) and intolerance to cold are NOT signs and symptoms of thyroid storm. All the other options are very typical signs and symptoms of thyroid storm.

3. The health care provider prescribes levothyroxine for a patient with hypothyroidism. After teaching regarding this drug, the nurse determines that further instruction is needed when the patient says a. "I can expect the medication dose may need to be adjusted." b. "I only need to take this drug until my symptoms are improved." c. "I can expect to return to normal function with the use of this drug." d. "I will report any chest pain or difficulty breathing to the doctor right away."

3. Correct answer: b Rationale: Levothyroxine is the drug of choice to treat hypothyroidism. The need for thyroid replacement therapy is usually lifelong.

78. Which medication order should the nurse question in the client diagnosed with untreated hypothyroidism? 1. Thyroid hormones. 2. Oxygen. 3. Sedatives. 4. Laxatives.

3. Sedatives. / 3. Untreated hypothyroidism is characterized by an increased susceptibility to the effects of most hypnotic and sedative agents; therefore, the nurse should question this medication. TEST-TAKING HINT: When a question asks which order the nurse should question, three of the options are medications the nurse expects to administer to the client. Sometimes saying, "The nurse administers this medication," may help the test taker select the correct answer.

30. A patient with hypoparathyroidism resulting from surgical treatment of hyperparathyroidism is preparing for discharge. What should the nurse teach the patient? a. Milk and milk products should be increased in the diet. b. Parenteral replacement of parathyroid hormone will be required for life. c. Calcium supplements with vitamin D can effectively maintain calcium balance. d. Bran and whole-grain foods should be used to prevent GI effects of replacement therapy

30. c. The hypocalcemia that results from PTH deficiency is controlled with calcium and vitamin D supplementation and possibly oral phosphate binders. Replacement with PTH is not used because of antibody formation to PTH, the need for parenteral administration, and cost. Milk products, although good sources of calcium, also have high levels of phosphate, which reduce calcium absorption. Whole grains and foods containing oxalic acid also impair calcium absorption

79. Which statement made by the client makes the nurse suspect the client is experiencing hyperthyroidism? 1. "I just don't seem to have any appetite anymore." 2. "I have a bowel movement about every three (3) to four (4) days." 3. "My skin is really becoming dry and coarse." 4. "I have noticed all my collars are getting tighter."

4. "I have noticed all my collars are getting tighter." / 4. The thyroid gland (in the neck) enlarges as a result of the increased need for thyroid hormone production; an enlarged gland is called a goiter. TEST-TAKING HINT: If the test taker does not know the answer, sometimes thinking about the location of the gland or organ causing the problem may help the test taker select or rule out specific options.

The nurse is performing an assessment on a client with a diagnosis of hyperthyroidism. Which assessment finding should the nurse expect to note in this client? Dry skin Bulging eyeballs Periorbital edema Coarse facial features

Bulging eyeballs Rationale: Hyperthyroidism is clinically manifested by goiter (increase in the size of the thyroid gland) and exophthalmos (bulging eyeballs). Other clinical manifestations include nervousness, fatigue, weight loss, muscle cramps, and heat intolerance. Additional signs found in this disorder include tachycardia; shortness of breath; excessive sweating; fine muscle tremors; thin, silky hair and thin skin; infrequent blinking; and a staring appearance.

82. The nurse is providing an in-service on thyroid disorders. One of the attendees asks the nurse, "Why don't the people in the United States get goiters as often?" Which statement by the nurse is the best response? 1. "It is because of the screening techniques used in the United States." 2. "It is a genetic predisposition rare in North Americans." 3. "The medications available in the United States decrease goiters." 4. "Iodized salt helps prevent the development of goiters in the United States."

4. "Iodized salt helps prevent the development of goiters in the United States." / 4. Almost all of the iodine entering the body is retained in the thyroid gland. A deficiency in iodine will cause the thyroid gland to work hard and enlarge, which is called a goiter. Goiters are commonly seen in geographical regions having an iodine deficiency. Most table salt in the United States has iodine added.

522. The nurse provides instructions to a client who is taking levothyroxine. The nurse should tell the client to take the medication in which way? 1. With food 2. At lunchtime 3. On an empty stomach 4. At bedtime with a snack

522. Answer: 3 Rationale: Oral doses of levothyroxine should be taken on an empty stomach to enhance absorption. Dosing should be done in the morning before breakfast. Test-Taking Strategy: Note that options 1, 2, and 4 are comparable or alike in that these options address administering the medication with food.

The nursing instructor asks a nursing student to identify the risk factors associated with the development of thyrotoxicosis. The student demonstrates understanding of the risk factors by identifying an increased risk for thyrotoxicosis in which client? A client with hypothyroidism A client with Graves' disease who is having surgery A client with diabetes mellitus scheduled for a diagnostic test A client with diabetes mellitus scheduled for debridement of a foot ulcer

A client with Graves' disease who is having surgery Rationale: Thyrotoxicosis usually is seen in clients with Graves' disease in whom the symptoms are precipitated by a major stressor. This complication typically occurs during periods of severe physiological or psychological stress such as trauma, sepsis, delivery, or major surgery. It also must be recognized as a potential complication after thyroidectomy. The client conditions in the remaining options are not associated with thyrotoxicosis.

The nurse is preparing to care for a client after parathyroidectomy. The nurse should plan for which action for this client? Maintain an endotracheal tube for 24 hours. Administer a continuous mist of room air or oxygen. Place the client in a flat position with the head and neck immobilized. Use only a rectal thermometer for temperature measurement.

Administer a continuous mist of room air or oxygen. Rationale: Humidification of air or oxygen helps to liquefy mucous secretions and promotes easier breathing after parathyroidectomy. Pooling of thick mucous secretions in the trachea, bronchi, and lungs will cause respiratory obstruction. The client will not necessarily have an endotracheal tube in place. Tympanic temperatures can be taken. Semi-Fowler's position is the position of choice to assist in lung expansion and prevent edema. Rectal temperatures only are not required.

A 33-year-old female client is admitted to the hospital with a tentative diagnosis of Graves' disease. Which symptom related to the menstrual cycle would the client be most likely to report during the initial assessment? Amenorrhea Menorrhagia Metrorrhagia Dysmenorrhea

Amenorrhea Rationale: Amenorrhea or a decreased menstrual flow occurs in the client with Graves' disease. Menorrhagia, metrorrhagia, and dysmenorrhea are also disorders related to the female reproductive system; however, they are not typical manifestations of Graves' disease.

The nurse is assessing a client who has a diagnosis of goiter. Which should the nurse expect to note during the assessment of the client? An enlarged thyroid gland The presence of heart damage Client complaints of chronic fatigue Client complaints of slow wound healing

An enlarged thyroid gland Rationale: An enlarged thyroid gland develops in the client with goiter because of an excessive amount of thyroxine in the thyroid gland. Heart damage occurs with selenium deficiency. In addition, heart damage would not likely be noted during the nursing assessment. Further diagnostic tests in addition to the assessment would be necessary to determine heart damage. Chronic fatigue occurs with iron deficiency. Slow wound healing occurs with zinc deficiency.

The nurse is developing a plan of care for a client who is scheduled for a thyroidectomy. The nurse focuses on psychosocial needs, knowing that which is likely to occur in the client? Infertility Gynecomastia Sexual dysfunction Body image changes

Body image changes Rationale: Because of the location of the incision in the neck area, many clients are afraid of thyroid surgery for fear of having a visible large scar postoperatively. Having all or part of the thyroid gland removed will not cause the client to experience gynecomastia. Sexual dysfunction and infertility could occur if the entire thyroid is removed and the client is not placed on thyroid replacement medications.

A client has returned to the nursing unit after a thyroidectomy. The nurse notes that the client is complaining of tingling sensations around the mouth, fingers, and toes. On the basis of these findings, the nurse should next assess the results of which serum laboratory study? Sodium Calcium Potassium Magnesium

Calcium Rationale: After surgery on the thyroid gland, the client may experience a temporary calcium imbalance. This is due to transient malfunction of the parathyroid glands. The nurse also would assess for Chvostek's and Trousseau's signs. The correct treatment is administration of calcium gluconate or calcium lactate. The remaining options are unrelated to the client's complaints.

12. A patient with hypothyroidism is having pain 6 on 1-10 scale in the right hip due to recent hip surgery. Which of the following medications are NOT appropriate for this patient? Select all that apply: A. Fentanyl B. Tylenol C. Morphine D. Dilaudid

The answers are A, C, and D. Patients who have hypothyroidism are very sensitive to narcotics and should take NON-NARCOTICS for pain relief. Fentanyl, Morphine, and Dilaudid are all narcotics, whereas, Tylenol is not.

The nurse is caring for Michaela who just returned to the post-op unit following a thyroidectomy. What cue is the nurse most concerned about? A. The patient complains of increased thirst. B. The patient reports a sore throat when swallowing. C. The patient needs help supporting her head when moving in bed. D. The patient makes harsh, vibratory sounds when breathing.

D. The patient makes harsh, vibratory sounds when breathing.

A nurse is caring for a client who had a thyroidectomy 1 day ago. Which client laboratory data should the nurse identify as a possible complication of thyroid surgery? Increased serum sodium level Increased serum glucose level Decreased serum calcium level Decreased serum albumin level

Decreased serum calcium level Rationale: Hypocalcemia may occur if the parathyroid glands are removed or damaged or if their blood supply is impaired during thyroid surgery, resulting in decreased parathyroid hormone (PTH) levels and leading to decreased serum calcium levels. Serum sodium, albumin, and glucose levels are not affected by thyroid surgery.

The nurse is reviewing the shift assignments and notes one of the clients has Grave's disease. The nurse knows the client with Grave's disease could potentially exhibit which of the following? Epistaxis Expothalmos Esotropia Exogeny

Expothalmos Expothalmos is bulging of the eyes and is a common sign of Grave's disease, which causes hyperthyroidism.

The nurse is monitoring a client with Graves' disease for signs of thyrotoxic crisis (thyroid storm). Which signs or symptoms, if noted in the client, will alert the nurse to the presence of this crisis? Fever and tachycardia Pallor and tachycardia Agitation and bradycardia Restlessness and bradycardia

Fever and tachycardia Rationale: Thyrotoxic crisis (thyroid storm) is an acute, potentially life-threatening state of extreme thyroid activity that represents a breakdown in the body's tolerance to a chronic excess of thyroid hormones. The clinical manifestations include fever with temperatures greater than 100º F, severe tachycardia, flushing and sweating, and marked agitation and restlessness. Delirium and coma can occur.

8. A patient is prescribed Fosamax (Alendronate). The patient is about to be discharged and you observe the patient taking the medication. Which of the following findings requires you to re-educate the patient on how to take this medication? A. The patient takes the medication on an empty stomach. B. The patient takes the medication with water. C. The patient sits up for 10 minutes after taking the medication. D. The patient waits 30 minutes after taking Fosamax before taking the prescribed vitamins and antacids.

Fosamax should be taken on an empty stomach, with a full glass of water, with no other medications for at least 30 minutes (especially antacids or vitamins), and the patient should sit-up for 30 minutes after taking it. Fosamax can cause serious throat and stomach ulcers and taking it with a full glass of water and sitting-up afterwards helps prevents this.

The nurse is caring for a client who has been poorly managing their Graves' disease. Which additional factor would cause the nurse major concern? Taking iron pills Diagnosis of COPD Starting a new exercise program Getting pregnant

Getting pregnant Poorly controlled Graves' disease causes major complications during pregnancy. The fetus and baby can have a low birth weight, preterm birth, or still birth.

A nurse is caring for a client with a dysfunctional thyroid gland and is concerned that the client will exhibit a sign of thyroid storm. Which is an early indicator of this complication? Bradycardia Constipation Hyperreflexia Low-grade temperature

Hyperreflexia Rationale: Clinical manifestations of thyroid storm include a fever as high as 106º F, hyperreflexia, abdominal pain, diarrhea, dehydration rapidly progressing to coma, severe tachycardia, extreme vasodilation, hypotension, atrial fibrillation, and cardiovascular collapse.

A nurse is caring for a client with thyrotoxicosis who is at risk for the development of thyroid storm. To detect this complication, the nurse should assess for which sign or symptom? Bradycardia Constipation Hypertension Low-grade temperature

Hypertension Rationale: Thyroid storm is an acute, life-threatening condition that occurs in a client with uncontrollable hyperthyroidism. Clinical manifestations of thyroid storm include systolic hypertension, tachycardia, diarrhea, and a fever as high as 106º F. Other manifestations include abdominal pain, dehydration, extreme vasodilation, stupor rapidly progressing to coma, atrial fibrillation, and cardiovascular collapse. Bradycardia, constipation, and low-grade temperature are not a part of the clinical picture in thyroid storm.

A client has begun medication therapy with propylthiouracil. The nurse should assess the client for which condition as an adverse effect of this medication? Joint pain Renal toxicity Hyperglycemia Hypothyroidism

Hypothyroidism Rationale: Propylthiouracil is prescribed for the treatment of hyperthyroidism. Excessive dosing with this agent may convert a hyperthyroid state to a hypothyroid state. If this occurs, the dosage should be reduced. Temporary administration of thyroid hormone may be required to treat the hypothyroid state. Propylthiouracil is not used for relief of joint pain. It does not cause renal toxicity or hyperglycemia.

The nurse is caring for a client after thyroidectomy. The client expresses concern about the postoperative voice hoarseness she is experiencing and asks if the hoarseness will subside. The nurse should provide the client with which information? It indicates nerve damage. The hoarseness is permanent. It is normal during this time and will subside. It will worsen before it subsides, which may take 6 months.

It is normal during this time and will subside. Rationale: Hoarseness in the postoperative period usually is the result of laryngeal pressure or edema and will resolve within a few days. The client should be reassured that the effects are transitory. The other options are incorrect.

A nurse is assessing the status of a client who returned to the surgical nursing unit after a parathyroidectomy procedure. The nurse would place highest priority on which assessment finding? Laryngeal stridor Difficulty voiding Mild incisional pain Absence of bowel sounds

Laryngeal stridor Rationale: During the early postoperative period, the nurse carefully observes the client for signs of bleeding, which may cause swelling and compression of adjacent tissues. Laryngeal stridor results from compression of the trachea and is a harsh, high-pitched sound heard on inspiration and expiration. Laryngeal stridor is an acute emergency, necessitating immediate attention to avoid complete obstruction of the airway. The other options describe usual postoperative problems that are not life threatening.

A client is admitted to an emergency department, and a diagnosis of myxedema coma is made. Which action should the nurse prepare to carry out initially? Warm the client. Maintain a patent airway. Administer thyroid hormone. Administer fluid replacement.

Maintain a patent airway. Rationale: Myxedema coma is a rare but serious disorder that results from persistently low thyroid production. Coma can be precipitated by acute illness, rapid withdrawal of thyroid medication, anesthesia and surgery, hypothermia, and the use of sedatives and opioid analgesics. In myxedema coma, the initial nursing action is to maintain a patent airway. Oxygen should be administered, followed by fluid replacement, keeping the client warm, monitoring vital signs, and administering thyroid hormones by the intravenous route.

A nurse is reviewing the primary health care provider's prescriptions for a client diagnosed with hypothyroidism. Which medication prescription should the nurse question and verify? Acetaminophen Docusate sodium Morphine sulfate Levothyroxine sodium

Morphine sulfate Rationale: Medications are administered very cautiously to the client with hypothyroidism because of altered metabolism and excretion and depressed metabolic rate and respiratory status. Morphine sulfate would further depress bodily functions. Hormone replacement with levothyroxine sodium, a thyroid hormone, is a component of therapy. Stool softeners, such as docusate sodium, are prescribed to prevent constipation. Acetaminophen can be taken.

7. A patient is recovery from a parathyroidectomy. Which of the following findings causes concern and requires nursing intervention? A. The patient is in Semi-Fowler's position. B. The patient's calcium level is 8.9 mg/dL. C. The patient's voice is hoarse. D. The patient is drowsy but arouses to name.

Patients who've had a parathyroidectomy are at risk for laryngeal nerve damage. Therefore, the nurse should monitor the patient for signs and symptoms of this which would include a hoarse voice, difficulty swallowing, or speaking. The nurse should intervene by notifying the physician.

The nurse is caring for a client with a new diagnosis of hypothyroidism. Which clinical manifestations might the nurse expect to note on examination of this client? Select all that apply. Irritability Periorbital edema Coarse, brittle hair Slow or slurred speech Abdominal distention Soft, silky, thinning hair

Periorbital edema Coarse, brittle hair Slow or slurred speech Abdominal distention Rationale: The manifestations of hypothyroidism are the result of decreased metabolism from low levels of thyroid hormones. The client may exhibit skin manifestations, such as coarse, brittle hair; thick, brittle nails; coarse, scaly skin; delayed wound healing; periorbital edema; and face puffiness. Neuromuscular manifestations include lethargy, slow or slurred speech, and impaired memory. Gastrointestinal manifestations include complaints of constipation, weight gain, and abdominal distention. Irritability and soft, silky, thinning hair on the scalp are manifestations of hyperthyroidism.

A primary health care provider has prescribed propylthiouracil for a client with hyperthyroidism. The nurse recalls that first-line treatment calls for methimazole for medication therapy. The nurse should question the client about her past medical history, specifically regarding which condition? Pregnancy Renal failure Prolonged QT interval Adverse reaction to levothyroxine

Pregnancy Rationale: Methimazole and propylthiouracil are both used to treat hyperthyroidism. Methimazole is considered first-line treatment; however, this medication cannot be used for clients who are in their first trimester of pregnancy, have had a previous adverse reaction to methimazole, or need rapid reduction of symptoms. Renal failure, prolonged QT interval, and adverse reaction to levothyroxine are not related to contraindications for methimazole.

A client has been brought to the emergency department suffering from thyroid storm. Which of the following signs or symptoms would the nurse expect to see in this situation? Periorbital edema Swelling of the hands, lips, and tongue Generalized rash over the entire body Rapid heart rate and fever

Rapid heart rate and fever Thyroid storm is a medical emergency in which thyroid hormone levels jump too high. The client typically develops a rapid heart rate, hypertension, irritability, tremors, and rapidly rising body temperature. The client should be seen in the emergency department if this condition develops for treatment with antithyroid medication, beta blockers, glucocorticoids and iodides.

3. A patient hospitalized with hypoparathyroidism is about to order lunch. Which food selection is best for this patient based on their dietary needs at this time? A. Baked chicken, green beans, and boiled potatoes B. Broccoli salad, cottage cheese, and peaches C. Roast beef, carrots, and pinto beans D. Hamburger, fries, and sorbet

Remember green leafy vegetables and cheeses are high in calcium. Therefore, B is the correct answer. All the other options are high in either protein, carbs, or fiber.

A nurse is caring for a client who has just returned from a thyroidectomy. What would be the most appropriate position for this client? Semi-Fowler's Orthopneic Supine Prone

Semi-Fowler's Semi-Fowler's would be the best position after this surgery. The patient is sitting upright, able to breathe well, and no pressure is being placed on the incision.

7. A patient is started on Tapazole (Methimazole) for treatment of Grave's Disease. Which statement by the patient indicates they understood your teaching about this medication? A. "If I experience fast heart, excessive sweating, or fever, I will notify the doctor immediately because I may be experiencing toxicity of the medication." B. "I know it may take a while before I feel relief of symptoms, therefore, I will never abruptly stop taking my medication." C. "This medication can cause high blood glucose." D. "I will make sure my diet is rich in foods containing iodine."

The answer B. Toxicity of Tapazole (an antithyroid medication) would include bradycardia, hypothermia, hypotension (signs and symptoms of HYPOthyroidism) etc. This medication is not known to cause high blood glucose, and the patient should avoid foods high in iodine because this causes an increased production of thyroid hormones.

5. Which of the following foods below should a patient experiencing a thyroid storm avoid? Select all that apply: A. Shrimp B. Milk C. Hard boiled eggs D. Seaweed (Kelp) E. Broccoli F. Peas

The answer is A, B, C, D. Foods high in iodine are seafoods like shrimp, seaweed, and dairy/eggs.

9. A patient is diagnosed with hyperparathyroidism. Which of the following signs and symptoms would you NOT find in this patient? Select all that apply: A. Calcium level 6 mg/dL B. Bone fracture C. Positive Trousseau's Sign D. Tingling and numbness of lips and fingers E. Calcium level of 15 mg/dL F. Phosphate level 1.2 G. Renal calculi

The answers are A, C, and D. This question wants to know what you will NOT see in hyperparathyroidism. So, you will need to select the options that won't happen. A patient with HYPERparathyroidism will NOT have a calcium level of 6mg/dL (it will be high), positive Trousseu's sign (this happens in cases of LOW calcium) along with tingling/numbnes of lips/fingers. Therefore, you can expect the patient to present with signs and symptoms of HYPERcalcemia. They will have a high calcium level and low phosphate level. A normal calcium level is 8.9 to 10 mg/dL and phosphate level is 2.7 to 4.5 mg/dL. Therefore, the patient would have possible bone fractures, calcium level greater than 10 mg/dL, hypophosphatemia, and possible renal calculi (kidney stone). All the other options are present in HYPOparathyroidism.

The nurse is monitoring a client for signs of hypocalcemia after thyroidectomy. Which sign or symptom, if noted in the client, would most likely indicate the presence of hypocalcemia? Bradycardia Flaccid paralysis Tingling around the mouth Absence of Chvostek's sign

Tingling around the mouth Rationale: After thyroidectomy the nurse assesses the client for signs of hypocalcemia and tetany. Early signs include tingling around the mouth and in the fingertips, muscle twitching or spasms, palpitations or arrhythmias, and Chvostek's and Trousseau's signs. Bradycardia, flaccid paralysis, and absence of Chvostek's sign are not signs of hypocalcemia.

The nurse is preparing for a client's postoperative return to the unit after a parathyroidectomy procedure. The nurse should ensure that which piece of medical equipment is at the client's bedside? Cardiac monitor Tracheotomy set Intermittent gastric suction device Underwater seal chest drainage system

Tracheotomy set Rationale: Respiratory distress caused by hemorrhage and swelling and compression of the trachea is a paramount concern for the nurse managing the care of a postoperative client who has had a parathyroidectomy. An emergency tracheotomy set is routinely placed at the bedside of the client who has undergone this type of surgery in anticipation of this complication. The items in the remaining options are not specifically needed with this surgical procedure.

The nurse has provided dietary instructions to a client with a diagnosis of hypoparathyroidism. The nurse should instruct the client that it is acceptable to include which item in the diet? Fish Cereals Vegetables Meat and poultry

Vegetables Rationale: The client with hypoparathyroidism is instructed to follow a calcium-rich diet and to restrict the amount of phosphorus in the diet. Vegetables are allowed in the diet. The client should limit meat, poultry, fish, eggs, cheese, and cereals.

A 36-year-old client must undergo a total thyroidectomy for treatment of thyroid cancer. Which of the following has been shown as a complication of this type of surgery? Injury to the clavicle Jugular vein distention Voice changes Hypercalcemia

Voice changes Total thyroidectomy involves the removal of the thyroid gland from the neck. It may be done as a response to thyroid cancer or in cases of a goiter or thyroid nodules when the gland is no longer functional. Because of the location of the thyroid, the client is at risk of voice changes associated with the surgery if the vocal cords are damaged during the surgical procedure. The nurse monitors for dysphonia and a high-pitched voice postoperatively.

A multidisciplinary health care team is developing a plan of care for a client with hyperparathyroidism. The nurse should include which priority intervention in the plan of care? Describe the use of loperamide. Restrict fluids to 1000 mL per day. Walk down the hall for 15 minutes 3 times a day. Describe the administration of aluminum hydroxide gel.

Walk down the hall for 15 minutes 3 times a day. Rationale: Mobility of the client with hyperparathyroidism should be encouraged as much as possible because of the calcium imbalance that occurs in this disorder and the predisposition to the formation of renal calculi. Fluids should not be restricted. Discussing the use of medications is not the priority with this client.

The nurse is caring for a client who is scheduled to have a thyroidectomy and provides instructions to the client about the surgical procedure. Which client statement indicates an understanding of the nurse's instructions? "I expect to experience some tingling of my toes, fingers, and lips after surgery." "I will definitely have to continue taking antithyroid medications after this surgery." "I need to place my hands behind my neck when I have to cough or change positions." "I need to turn my head and neck front, back, and laterally every hour for the first 12 hours after surgery."

"I need to place my hands behind my neck when I have to cough or change positions." Rationale: The client is taught that tension needs to be avoided on the suture line; otherwise hemorrhage may develop. One way of reducing incisional tension is to teach the client how to support the neck when coughing or being repositioned. Likewise, during the postoperative period the client should avoid any unnecessary movement of the neck. That is why sandbags and pillows frequently are used to support the head and neck. Any postoperative tingling in the fingers, toes, and lips probably is due to injury to the parathyroid gland during surgery, resulting in hypocalcemia. These signs and symptoms need to be reported immediately. Removal of the thyroid does not mean that the client will be taking antithyroid medications postoperatively. Thyroid replacement medications are necessary.

The nurse is teaching a client with hyperparathyroidism how to manage the condition at home. Which response by the client indicates the need for additional teaching? "I should consume less than 1 liter of fluid per day." "I should use my treadmill or go for walks daily." "I should follow a moderate-calcium, high-fiber diet." "My alendronate helps keep calcium from coming out of my bones."

"I should consume less than 1 liter of fluid per day." Rationale: In hyperparathyroidism, clients experience excess parathyroid hormone (PTH) secretion. A role of PTH in the body is to maintain serum calcium homeostasis. When PTH levels are high, there is excess bone resorption (calcium is pulled from the bones). In clients with elevated serum calcium levels, there is a risk of nephrolithiasis. One to two liters of fluids daily should be encouraged to protect the kidneys and decrease the risk of nephrolithiasis. Moderate physical activity, particularly weight-bearing activity, minimizes bone resorption and helps protect against pathological fracture. Walking, as an exercise, should be encouraged in the client with hyperparathyroidism. Even though serum calcium is already high, clients should follow a moderate-calcium diet, because a low-calcium diet will surge PTH. Calcium causes constipation, so a diet high in fiber is recommended. Alendronate is a bisphosphate that inhibits bone resorption. In bone resorption, bone is broken down and calcium is deposited into the serum.

16. A patient is admitted to the hospital with acute thyrotoxicosis. On physical assessment of the patient, what should the nurse expect to find? a. Hoarseness and laryngeal stridor b. Bulging eyeballs and dysrhythmias c. Elevated temperature and signs of heart failure d. Lethargy progressing suddenly to impairment of consciousness

16. c. A thyroid storm results in marked manifestations of hyperthyroidism, with severe tachycardia, heart failure, shock, hyperthermia, agitation delirium, seizures, abdominal pain, vomiting, diarrhea, and coma. Although exophthalmos may be present in the patient with Graves' disease, it is not a significant factor in thyrotoxic crisis. Hoarseness and laryngeal stridor are characteristic of the tetany of hypoparathyroidism and lethargy progressing to coma is characteristic of myxedema coma, a complication of hypothyroidis

80. The 68-year-old client diagnosed with hyperthyroidism is being treated with radioactive iodine therapy. Which interventions should the nurse discuss with the client? 1. Explain it will take up to a month for symptoms of hyperthyroidism to subside. 2. Teach the iodine therapy will have to be tapered slowly over one (1) week. 3. Discuss the client will have to be hospitalized during the radioactive therapy. 4. Inform the client after therapy the client will not have to take any medication.

1. Explain it will take up to a month for symptoms of hyperthyroidism to subside. / 1. Radioactive iodine therapy is used to destroy the overactive thyroid cells. After treatment, the client is followed closely for three (3) to four (4) weeks until the euthyroid state is reached. TEST-TAKING HINT: Some questions require the test taker to be knowledgeable of the information, especially medical treatments, and there are no specific hints to help the test taker answer the question.

18. Which characteristics most accurately describe the use of RAI (select all that apply)? a. Decreases release of thyroid hormones b. Often causes hypothyroidism over time c. Blocks peripheral conversion of T4 to T3 d. Treatment of choice in nonpregnant adults e. Often used with iodine to produce euthyroid before surgery f. Decreases thyroid hormone secretion by damaging thyroid gland

18. b, d, f. RAI causes hypothyroidism over time by damaging thyroid tissue, which decreases thyroid hormone secretion and is the treatment of choice for nonpregnant adults. Potassium iodine decreases the release of thyroid hormones and decreases the size of the thyroid gland preoperatively. Propylthiouracil blocks peripheral conversion of T4 to T3 and may be used with iodine to produce a euthyroid state before surg

19. What preoperative instruction should the nurse give to the patient scheduled for a subtotal thyroidectomy? a. How to support the head with the hands when turning in bed b. Coughing should be avoided to prevent pressure on the incision c. Head and neck will have to remain immobile until the incision heals d. Any tingling around the lips or in the fingers after surgery is expected and temporary

19. a. To prevent strain on the suture line postoperatively, the patient's head must be manually supported while turning and moving in bed, but range-of-motion exercises for the head and neck are also taught preoperatively to be gradually implemented after surgery. There is no contraindication for coughing and deep breathing, and these should be carried out postoperatively. Tingling around the lips or fingers is a sign of hypocalcemia, which may occur if the parathyroid glands are inadvertently removed during surgery. This sign should be reported immediately.

26. A patient who recently had a calcium oxalate renal stone had a bone density study, which showed a decrease in her bone density. What endocrine problem could this patient have? a. SIADH c. Cushing syndrome b. Hypothyroidism d. Hyperparathyroidism

26. d. The patient with hyperparathyroidism may have calcium nephrolithiasis, skeletal pain, decreased bone density, psychomotor retardation, or cardiac dysrhythmias among other manifestations. The other endocrine problems would not be related to calcium kidney stones or decreased bone density.

28. A patient has been diagnosed with hypoparathyroidism. What manifestations should the nurse expect to observe (select all that apply)? a. Skeletal pain b. Dry, scaly skin c. Personality changes d. Abdominal cramping e. Cardiac dysrhythmias f. Muscle spasms and stiffness

28. b, c, d, e, f. In hypoparathyroidism the patient has inadequate circulating parathyroid hormone (PTH) that leads to hypocalcemia from the inability to maintain serum calcium levels. With hypocalcemia there is muscle stiffness and spasms, which can lead to cardiac dysrhythmias and abdominal cramps. There can also be personality and visual changes and dry, scaly skin

75. The client diagnosed with hypothyroidism is prescribed the thyroid hormone levothyroxine (Synthroid). Which assessment data indicate the medication has been effective? 1. The client has a three (3)-pound weight gain. 2. The client has a decreased pulse rate. 3. The client's temperature is WNL. 4. The client denies any diaphoresis.

3. The client's temperature is WNL. / 3. The client with hypothyroidism frequently has a subnormal temperature, so a temperature WNL indicates the medication is effective. TEST-TAKING HINT: One way of determining the effectiveness of medication is to determine if the signs/symptoms of the disease are no longer noticeable.

521. The nurse is monitoring a client receiving levothyroxine sodium for hypothyroidism. Which findings indicate the presence of a side effect associated with this medication? Select all that apply. 1. icon01-9780323358415Insomnia 2. icon01-9780323358415Weight loss 3. icon01-9780323358415Bradycardia 4. icon01-9780323358415Constipation 5. icon01-9780323358415Mild heat intolerance

521. Answer: 1, 2, 5 Rationale: Insomnia, weight loss, and mild heat intolerance are side effects of levothyroxine sodium. Bradycardia and constipation are not side effects associated with this medication, and rather are associated with hypothyroidism, which is the disorder that this medication is prescribed to treat. Test-Taking Strategy: Focus on the subject, side effects of levothyroxine. Thinking about the pathophysiology of hypothyroidism and the action of the medication will assist you in determining that insomnia, weight loss, and mild heat intolerance are side effects of thyroid hormones.

523. The nurse should tell the client who is taking levothyroxine to notify the primary health care provider (PHCP) if which problem occurs? 1. Fatigue 2. Tremors 3. Cold intolerance 4. Excessively dry skin

523. Answer: 2 Rationale: Excessive doses of levothyroxine can produce signs and symptoms of hyperthyroidism. These include tachycardia, chest pain, tremors, nervousness, insomnia, hyperthermia, extreme heat intolerance, and sweating. The client should be instructed to notify the PHCP if these occur. Options 1, 3, and 4 are signs of hypothyroidism. Test-Taking Strategy: Focus on the subject, the need to notify the PHCP. Recall the symptoms associated with hypothyroidism, the purpose of administering levothyroxine, and the effects of the medication. Options 1, 3, and 4 are symptoms related to hypothyroidism.

527. The client with hyperparathyroidism is taking alendronate. Which statements by the client indicate understanding of the proper way to take this medication? Select all that apply. 1. icon01-9780323358415"I should take this medication with food." 2. icon01-9780323358415"I should take this medication at bedtime." 3. icon01-9780323358415"I should sit up for at least 30 minutes after taking this medication." 4. icon01-9780323358415"I should take this medication first thing in the morning on an empty stomach." 5. icon01-9780323358415"I can pick a time to take this medication that best fits my lifestyle as long as I take it at the same time each day."

527. Answer: 3, 4 Rationale: Alendronate is a bisphosphonate used in hyperparathyroidism to inhibit bone loss and normalize serum calcium levels. Esophagitis is an adverse effect of primary concern in clients taking alendronate. For this reason the client is instructed to take alendronate first thing in the morning with a full glass of water on an empty stomach, not to eat or drink anything else for at least 30 minutes after taking the medication, and to remain sitting upright for at least 30 minutes after taking it. Test-Taking Strategy: Focus on the subject, the correct method to take alendronate. Recall that the primary concern with alendronate is esophagitis. Eliminate options 1 and 2, since taking with food and taking at bedtime will each place the client at increased risk of reflux. Eliminate option 5 because alendronate should be taken first thing in the morning on an empty stomach.

2. As the nurse educating the patient about Grave's Disease, which of the following statements by the patient ensures the patient understood the education about their condition? A. "I could experience myxedema coma, which is life-threatening, if I abruptly stop taking my antithyroid medication." B. "Grave's disease is due to an excessive amount of thyroid hormone in the body." C. "I will be sure to eat a lot of kelp because it helps with decreasing thyroid hormone levels." D. "If I have pain I will only take aspirin."

Answer is B. Option A is wrong because myxedema coma is a complication of HYPOthyroidism, and if the patient abruptly stops taking their antithyroid medication they are at risk for THYROID STORM. Kelp (seaweed) and Aspirin should be avoided in Grave's disease because they both cause increased T3 and T4 production.

A nurse is caring for a client who has had thyroid surgery. The client's calcium result has returned at 7.5 mg/dL. Which nursing interventions are the most appropriate based on this calcium level? Select all that apply. Assess for signs of hypocalcemia Assess for signs of hypercalcemia Implement seizure precautions Administer intravenous calcium gluconate Administer intravenous furosemide

Assess for signs of hypocalcemia Implement seizure precautions Administer intravenous calcium gluconate The nurse should assess for signs of hypocalcemia, which include numbness or tingling of the face and extremities, muscle cramps and twitching, seizures, cardiac dysrhythmias, and a positive Chvostek's sign. Chvosteks' sign occurs when you tap the facial nerve and the facial muscle twitches on the respective side. The client who has had thyroid surgery is at risk for hypocalcemia due to possible trauma to the parathyroid. Normal calcium levels are 8.4-10.2 mg/dL, so a level of 7.5 mg/dL indicates hypocalcemia. Calcium gluconate is given to increase the serum calcium level. We would not wait to treat this level, since we know the client is likely not going to be able to self-correct, and the consequences of hypocalcemia can be life-threatening.

6. You are providing discharge teaching to a patient who is prescribed calcium supplements with vitamin D for treatment of hypoparathyroidism. Which of the following statements by the patient warrants you to re-educate the patient on how they should take this medication? A. "I will also make sure I eat foods rich in calcium, such as dairy and green leafy vegetables while I'm taking this medication." B. "A side effect of this medication is constipation. Therefore, I should drink plenty of fluids." C. "I will take my calcium supplements in the morning when I take my Synthroid." D. All the statements above are correctly stated by the patient.

Calcium supplements should NOT be taken at the same time as iron or thyroid hormones (Synthroid) because it affects absorption. Therefore, this medications should be taken at separate times.

The nurse should include which interventions in the plan of care for a client with hypothyroidism? Select all that apply. Provide a cool environment for the client. Instruct the client to consume a high-fat diet. Instruct the client about thyroid replacement therapy. Encourage the client to consume fluids and high-fiber foods in the diet. Inform the client that iodine preparations will be prescribed to treat the disorder. Instruct the client to contact the primary health care provider (PHCP) if episodes of chest pain occur.

Instruct the client about thyroid replacement therapy. Encourage the client to consume fluids and high-fiber foods in the diet. Instruct the client to contact the primary health care provider (PHCP) if episodes of chest pain occur. Rationale: The clinical manifestations of hypothyroidism are the result of decreased metabolism from low levels of thyroid hormone. Interventions are aimed at replacement of the hormone and providing measures to support the signs and symptoms related to decreased metabolism. The client often has cold intolerance and requires a warm environment. The nurse encourages the client to consume a well-balanced diet that is low in fat for weight reduction and high in fluids and high-fiber foods to prevent constipation. Iodine preparations may be used to treat hyperthyroidism. Iodine preparations decrease blood flow through the thyroid gland and reduce the production and release of thyroid hormone; they are not used to treat hypothyroidism. The client is instructed to notify the PHCP if chest pain occurs because it could be an indication of overreplacement of thyroid hormone.

A client with a recent history of total thyroidectomy has developed iatrogenic hypoparathyroidism. Which observed findings does the nurse determine are associated with the hypoparathyroidism? Select all that apply. Laryngospasm Nephrolithiasis Muscle weakness Positive Chvostek's sign Positive Trousseau's sign

Laryngospasm Positive Chvostek's sign Positive Trousseau's sign Rationale: Hypoparathyroidism is an uncommon condition associated with inadequate circulating parathyroid hormone (PTH). It is characterized by hypocalcemia resulting from a lack of PTH to maintain serum calcium levels. The most common cause is iatrogenic; for example, accidental removal of the parathyroid gland during neck surgery. Signs and symptoms of hypocalcemia include laryngospasm and positive Chvostek's and Trousseau's signs. The remaining options are incorrect.

The nurse has developed a postoperative plan of care for a client who had a thyroidectomy and documents that the client is at risk for developing an ineffective breathing pattern. Which nursing intervention should the nurse include in the plan of care? Maintain a supine position. Monitor neck circumference every 4 hours. Maintain a pressure dressing on the operative site. Encourage deep-breathing exercises and vigorous coughing exercises.

Monitor neck circumference every 4 hours. Rationale: After thyroidectomy, neck circumference is monitored every 2 hours to assess for the occurrence of postoperative edema. The client should be placed in an upright position to facilitate air exchange. A pressure dressing is not placed on the operative site because it may restrict breathing. The nurse should monitor the dressing closely and should loosen the dressing if necessary. The nurse should assist the client with deep-breathing exercises, but coughing is minimized to prevent tissue damage and stress to the incision.

A client has overactivity of the thyroid gland. The nurse should expect which finding? Weight gain Nutritional deficiencies Low blood glucose levels Increased body fat stores

Nutritional deficiencies Rationale: Although the client may experience an increased appetite with overactivity of the thyroid gland, food intake does not meet energy demands, and nutritional deficiencies can develop. Weight loss occurs as a result of the increased metabolic activity. Glucose tolerance is decreased, and the client experiences hyperglycemia. Overactivity of the thyroid gland also causes increased metabolism, including fat metabolism. This leads to decreased levels of fat in the bloodstream, including cholesterol, and decreased body fat stores.

A client with Graves' disease has exophthalmos and is experiencing photophobia. Which nursing action would best assist the client with these manifestations? Obtain dark glasses for the client. Lubricate the eyes with tap water every 2 to 4 hours. Administer methimazole every 8 hours around the clock. Instruct the client to avoid straining or heavy lifting because this effort can increase eye pressure.

Obtain dark glasses for the client. Rationale: Because photophobia (light intolerance) accompanies this disorder, wearing dark glasses is helpful in alleviating the problem. Tap water, which is hypotonic, could actually cause more swelling to the eye because it could pull fluid into the interstitial space. In addition, the client would be at risk for developing an eye infection because the solution is not sterile. Methimazole is a thyroid inhibitor, but medication therapy for Graves' disease does not help to alleviate the clinical manifestation of exophthalmos. There is no need to avoid straining or heavy lifting with exophthalmos.

3. A physician orders a patient in thyroid storm to be started on Inderal. What in the patient's health history causes the nurse to question the doctor's order? A. History of mental illness B. History of asthma C. History of tachycardia D. History of cancer

Patients with a history of asthma should not take Inderal (a beta blocker) because it can cause asthma exacerbation or bronchospasm. Therefore, the nurse should question this order.

During physical examination of a client, which finding is characteristic of hypothyroidism? Periorbital edema Flushed, warm skin Hyperactive bowel sounds Heart rate of 120 beats/min

Periorbital edema Rationale: Because cellular edema occurs in hypothyroidism, the client's appearance is changed. Nonpitting edema occurs, especially around the eyes and in the feet and hands. Knowing this should direct you to option 1. Flushed, warm skin; hyperactive bowel sounds; and tachycardia (heart rate >100 beats/min) are clinical manifestations of hyperthyroidism, which occurs as a result of excess thyroid hormone secretion, resulting in a hypermetabolic state.

The nurse is completing an assessment on a client who is being admitted for a diagnostic workup for primary hyperparathyroidism. Which client complaints would be characteristic of this disorder? Select all that apply. Polyuria Headache Bone pain Nervousness Weight gain

Polyuria Bone pain Rationale: The role of parathyroid hormone (PTH) in the body is to maintain serum calcium homeostasis. In hyperparathyroidism, PTH levels are high, which causes bone resorption (calcium is pulled from the bones). Hypercalcemia occurs with hyperparathyroidism. Elevated serum calcium levels produce osmotic diuresis and thus polyuria. This diuresis leads to dehydration (weight loss rather than weight gain). Loss of calcium from the bones causes bone pain. Options 2, 4, and 5 are not associated with hyperparathyroidism. Some gastrointestinal symptoms include anorexia, nausea, vomiting, and constipation.

The nurse caring for a client with a diagnosis of hypoparathyroidism reviews the laboratory results of blood tests for this client and notes that the calcium level is extremely low. The nurse should expect to note which finding on assessment of the client? Unresponsive pupils Positive Trousseau's sign Negative Chvostek's sign Hyperactive bowel sounds

Positive Trousseau's sign Rationale: Hypoparathyroidism is related to a lack of parathyroid hormone secretion or a decreased effectiveness of parathyroid hormone on target tissues. The end result of this disorder is hypocalcemia. When serum calcium levels are critically low, the client may exhibit Chvostek's and Trousseau's signs, which indicate potential tetany. The remaining options are not related to the presence of hypocalcemia.

14. Which of the following signs and symptoms causes concern and requires nursing intervention for a patient who recently had a thyroidectomy? A. Heart rate of 120, blood pressure 220/102, temperature 103.2 'F B. Heart rate of 35, blood pressure 60/43, temperature 95.3 'F C. Soft hair, irritable, diarrhea D. Constipation, drowsiness, goiter

The answer is A. A patient is at risk for experiencing thyroid storm after a thyroidectomy because of manipulation of the thryroid gland that could cause excessive T3 and T4 to enter into the bloodstream during removal of the gland. Therefore, heart rate of 120, blood pressure 220/102, temperature 103.2 'F are classic signs of thyroid storm and this requires nursing intervention.

16. Which of the following side effects are possible for a patient taking an anti-thyroid medication? A. Agranulocytosis and aplastic anemia B. Tachycardia C. Skin discoloration D. Joint pain and eczema

The answer is A: Agranulocytosis and aplastic anemia

5. You are performing discharge teaching with a patient who is going home on Synthroid. Which statement by the patient causes you to re-educate the patient about this medication? A. "I will take this medication at bedtime with a snack." B. "I will never stop taking the medication abruptly." C. "If I have palpitations, chest pain, intolerance to heat, or feel restless, I will notify the doctor." D. "I will not take this medication at the same time I take my Carafate."

The answer is A: Synthroid is best taken in the MORNING on an empty stomach. All the other statements are correct about taking Synthroid.

11. A patient is being educated on how to take their anti-thyroid medication. Which of the following statements are INCORRECT? A. "I will continue taking aspirin daily." B. "I will take this medication at the same time every day." C. "It may take a while before I notice that the medication is helping my condition." D. "I will avoid foods containing high levels of iodine."

The answer is A: The patient needs to be instructed NOT to take aspirin because it increases thyroid hormones. All the other statements are correct.

7. A patient is admitted with complaints of palpations, excessive sweating, and unable to tolerate heat. In addition, the patient voices concern about how her appearance has changed over the past year. The patient presents with protruding eyeballs and pretibial myxedema on the legs and feet. Which of the following is the likely cause of the patient's signs and symptoms? A. Thyroiditis B. Deficiency of iodine consumption C. Grave's Disease D. Hypothyroidism

The answer is C: Grave's Disease

The nurse is caring for a client after thyroidectomy. The nurse notes that calcium gluconate is prescribed for the client. The nurse determines that this medication has been prescribed for which purpose? To treat thyroid storm To prevent cardiac irritability To treat hypocalcemic tetany To stimulate release of parathyroid hormone

To treat hypocalcemic tetany Rationale: Hypocalcemia, resulting in tetany, can develop after thyroidectomy if the parathyroid glands are accidentally removed during surgery. Manifestations develop 1 to 7 days after surgery. If the client develops numbness and tingling around the mouth, fingertips, or toes; muscle spasms; or twitching, the primary health care provider is notified immediately. Calcium gluconate should be readily available in the nursing unit.

A nurse is caring for a client after a thyroidectomy. Which specific emergency equipment should the nurse have available as it relates to this procedure? Defibrillator Tracheostomy tray Dextrose 50% in water Normal saline for intravenous bolus

Tracheostomy tray Rationale: After thyroidectomy, airway obstruction, although not common, can occur. This is considered an emergency situation. If this develops, emergency management needs to occur, and oxygen, suction equipment, and a tracheostomy tray should be immediately available at the bedside. The other supplies are not necessary specifically for thyroidectomy.

a.Treatment of choice in non-pregnant adults b.Inhibit synthesis of thyroid hormones c.High incidence of post-treatment hypothyroidism d.Used to â vascularity e.Used to control sympathetic symptoms f.Destroys thyroid tissue, limiting thyroid hormone secretion. 1.Propylthiouracil (PTU) 2.Potassium iodine 3.Propranolol (Inderal) 4.Radioactive Iodine 131

a. 4 b. 1 c. 4 d. 2 e. 3 f. 4

4. Signs and Symptoms of myxedema coma include all of the following EXCEPT? Select all that apply: A. Fever B. Bradycardia C. Sodium level less than 135 D. Blood glucose level greater than 350 E. Hypothermia

A. Fever D. Blood glucose level greater than 350

4. A patient is receiving radioactive iodine as treatment for Grave's Disease. Which of the following are common side effects of the treatment? Select all that apply: A. Nausea B. Taste changes C. Excessive saliva D. Swollen salivary glands

A. Nausea B. Taste changes D. Swollen salivary glands

1. Which patient is most at risk for Thyroid Storm? A. A 60 year old female who reports not taking Synthroid regularly. B. A 45 year old male who has not been taking Tapazole as ordered and is experiencing diabetic ketoacidosis. C. A 6 year old with an allergy to iodine. D. A 25 year old female who is pregnant with her 4th child and is experiencing eczema.

Answer is B. The red flag in this option is "not been taking Tapazole" and is experiencing "DKA". This indicates the patient has hyperthyroidism (Tapazole is an antithyroid medication) and this already puts him at risk for thyroid storm. Then DKA is another added stress on the body that can send him into thyroid storm. All the other options are either incorrect or the patient is at risk for myxedema coma (a complication of HYPOTHYROIDISM).

A 40-year-old female client with recent thyroid surgery has been admitted to the hospital with a fever, tremors, profuse sweating and anxiety. Based on this client's symptoms, which drug class would be a priority to give to this client? Antianxiety medication Antipyretic medication Calcium channel blocker Antithyroid medication

Antithyroid medication Thyroid storm is a life-threatening condition that is caused by an excess production of thyroid hormones, which leads to a hypermetabolic state. The condition is also referred to as thyrotoxicosis. This can occur in the client with uncontrollable hyperthyroidism when the thyroid is manipulated during surgery, from severe infection or stress. The client in thyroid storm is given antithyroid medications, iodides, propranolol and/or glucocorticoids, vital signs are monitored and adequate ventilation should be maintained.

73. The client is diagnosed with hypothyroidism. Which signs/symptoms should the nurse expect the client to exhibit? 1. Complaints of extreme fatigue and hair loss. 2. Exophthalmos and complaints of nervousness. 3. Complaints of profuse sweating and flushed skin. 4. Tetany and complaints of stiffness of the hands.

1. Complaints of extreme fatigue and hair loss. / 73. 1. A decrease in thyroid hormone causes decreased metabolism, which leads to fatigue and hair loss. TEST-TAKING HINT: Often if the test taker does not know the specific signs/symptoms of the disease but knows the function of the system affected by the disease, some possible answers can be ruled out. Tetany and stiffness of the hands are related to calcium, the level of which is influenced by the parathyroid gland, not the thyroid gland; therefore, option "4" can be ruled out.

88. The client diagnosed with hyperthyroidism is complaining of being hot and cannot sit still. Which should the nurse do based on the assessment? 1. Continue to monitor the client. 2. Have the UAP take the client's vital signs. 3. Request an order for a sedative. 4. Insist the client lie down and rest.

1. Continue to monitor the client. / 1. The nurse should continue to monitor the client. The behavior is expected for a client with hyperthyroidism. TEST-TAKING HINT: The test taker must know expected symptoms for disease processes.

74. The nurse identifies the client problem "risk for imbalanced body temperature" for the client diagnosed with hypothyroidism. Which intervention should be included in the plan of care? 1. Discourage the use of an electric blanket. 2. Assess the client's temperature every two (2) hours. 3. Keep the room temperature cool. 4. Space activities to promote rest.

1. Discourage the use of an electric blanket. / 1. External heat sources (heating pads, electric or warming blankets) should be discouraged because they increase the risk of peripheral vasodilation and vascular collapse. TEST-TAKING HINT: The test taker must always know exactly what the question is asking. Option "4" can be ruled out because it does not address body temperature. If the test taker knows the normal function of the thyroid gland, this may help identify the answer; decreased metabolism will cause the client to be cold.

81. The nurse is teaching the client diagnosed with hyperthyroidism. Which information should be taught to the client? Select all that apply. 1. Notify the HCP if a three (3)-pound weight loss occurs in two (2) days. 2. Discuss ways to cope with the emotional lability. 3. Notify the HCP if taking over-the-counter medication. 4. Carry a medical identification card or bracelet. 5. Teach how to take thyroid medications correctly.

1. Notify the HCP if a three (3)-pound weight loss occurs in two (2) days. / 1. Weight loss indicates the medication may not be effective and will probably need to be increased. 2. Discuss ways to cope with the emotional lability. / 2. The client needs to know emotional highs and lows are secondary to hyperthyroidism. With treatment, this emotional lability will subside. 3. Notify the HCP if taking over-the-counter medication. / 3. Any over-the-counter medications (for example, alcohol-based medications) may negatively affect the client's hyperthyroidism or medications being used for treatment. 4. Carry a medical identification card or bracelet. / 4. This will help any HCP immediately know of the client's condition, especially if the client is unable to tell the HCP. TEST-TAKING HINT: This alternate-type question instructs the test taker to select all the interventions that apply. The test taker must read and evaluate each option as to whether it applies or not.

14. Which statement accurately describes Graves' disease? a. Exophthalmos occurs in Graves' disease. b. It is an uncommon form of hyperthyroidism. c. Manifestations of hyperthyroidism occur from tissue desensitization to the sympathetic nervous system. d. Diagnostic testing in the patient with Graves' disease will reveal an increased thyroid-stimulating hormone (TSH) level.

14. a. Exophthalmos or protrusion of the eyeballs may occur in Graves' disease from increased fat deposits and fluid in the orbital tissues and ocular muscles, forcing the eyeballs outward. Graves' disease is the most common form of hyperthyroidism. Increased metabolic rate and sensitivity of the sympathetic nervous system lead to the clinical manifestations. Thyroid-stimulating hormone (TSH) level is decreased in Graves' diseas

15. A patient with Graves' disease asks the nurse what caused the disorder. What is the best response by the nurse? a. "The cause of Graves' disease is not known, although it is thought to be genetic." b. "It is usually associated with goiter formation from an iodine deficiency over a long period of time." c. "Antibodies develop against thyroid tissue and destroy it, causing a deficiency of thyroid hormones." d. "In genetically susceptible persons, antibodies are formed that cause excessive thyroid hormone secretion."

15. d. In Graves' disease, antibodies to the TSH receptor are formed, attach to the receptors, and stimulate the thyroid gland to release triiodothyronine (T3), thyroxine (T4), or both, creating hyperthyroidism. The disease is not directly genetic, but individuals appear to have a genetic susceptibility to develop autoimmune antibodies. Goiter formation from insufficient iodine intake is usually associated with hypothyroidism.

17. What medication is used with thyrotoxicosis to block the effects of the sympathetic nervous stimulation of the thyroid hormones? a. Potassium iodine c. Propranolol (Inderal) b. Propylthiouracil d. Radioactive iodine (RAI)

17. c. The β-adrenergic blocker propranolol is usually used to block the sympathetic nervous system stimulation by thyroid hormones. Atenolol would be used with asthma or heart disease. Potassium iodine is used to prepare the patient for thyroidectomy or for treatment of thyrotoxic crisis to inhibit the synthesis of thyroid hormones. Propylthiouracil, an antithyroid medication, inhibits the synthesis of thyroid hormones. Radioactive iodine (RAI) therapy destroys thyroid tissue, which limits thyroid hormone secretion.

84. Which signs/symptoms should make the nurse suspect the client is experiencing a thyroid storm? 1. Obstipation and hypoactive bowel sounds. 2. Hyperpyrexia and extreme tachycardia. 3. Hypotension and bradycardia. 4. Decreased respirations and hypoxia.

2. Hyperpyrexia and extreme tachycardia. / 2. Hyperpyrexia (high fever) and heart rate above 130 beats per minute are signs of thyroid storm, a severely exaggerated hyperthyroidism. TEST-TAKING HINT: If the test taker does not have the knowledge to answer the question, the test taker should look at the options closely. Options "1," "3," and "4" all have signs/symptoms of "decrease"—hypoactive, hypotension, and hypoxia. The test taker should select the option that does not match.

77. The client is admitted to the intensive care department diagnosed with myxedema coma. Which assessment data warrant immediate intervention by the nurse? 1. Serum blood glucose level of 74 mg/dL. 2. Pulse oximeter reading of 90%. 3. Telemetry reading showing sinus bradycardia. 4. The client is lethargic and sleeps all the time.

2. Pulse oximeter reading of 90%. / 2. A pulse oximeter reading of less than 93% is significant. A 90% pulse oximeter reading indicates a Pao2 of approximately 60 on an arterial blood gas test; this is severe hypoxemia and requires immediate intervention. TEST-TAKING HINT: The words "warrant immediate intervention" means the test taker should select an option that is abnormal for the disease process or a life-threatening symptom.

92. The client is diagnosed with hypothyroidism. Which assessment data support this diagnosis? 1. The client's vital signs are: T 99.0, P 110, R 26, and BP 145.80. 2. The client complains of constipation and being constantly cold. 3. The client has an intake of 780 mL and output of 256 mL. 4. The client complains of a headache and has projectile vomiting.

2. The client complains of constipation and being constantly cold. / 2. All body processes slow as a result of decreased thyroid production. The client will be constipated, cold, have thicker skin, low temperature, and bradycardia. TEST-TAKING HINT: The test taker must know basic signs and symptoms. The word "hypo" in the name of the disease would help the test taker eliminate option "1."

20. Priority Decision: As a precaution for vocal cord paralysis from damage to the recurrent or superior laryngeal nerve during thyroidectomy surgery, what is the most important equipment to have in the room in case it is needed for this emergency situation? a. Tracheostomy tray c. IV calcium gluconate b. Oxygen equipment d. Paper and pencil for communication

20. a. A tracheostomy tray must be in the room to use if the emergency situation of vocal cord paralysis occurs from recurrent or superior laryngeal nerve damage or for laryngeal stridor from tetany. The oxygen equipment may be useful but will not improve oxygenation with vocal cord paralysis without a tracheostomy. IV calcium salts will be used if hypocalcemia occurs from parathyroid damage. The paper and pencil for communication may be helpful, especially if a tracheostomy is performed, but will not aid in emergency oxygenation of the patient

21. When providing discharge instructions to a patient who had a subtotal thyroidectomy for hyperthyroidism, what should the nurse teach the patient? a. Never miss a daily dose of thyroid replacement therapy. b. Avoid regular exercise until thyroid function is normalized. c. Use warm saltwater gargles several times a day to relieve throat pain. d. Substantially reduce caloric intake compared to what was eaten before surgery.

21. d. With the decrease in thyroid hormone postoperatively, calories must be reduced substantially to prevent weight gain. When a patient has had a subtotal thyroidectomy, thyroid replacement therapy is not given because exogenous hormone inhibits pituitary production of TSH and delays or prevents the restoration of thyroid tissue regeneration. Regular exercise stimulates the thyroid gland and is encouraged. Saltwater gargles are used for dryness and irritation of the mouth and throat following radioactive iodine therapy

22. What is a cause of primary hypothyroidism in adults? a. Malignant or benign thyroid nodules b. Surgical removal or failure of the pituitary gland c. Surgical removal or radiation of the thyroid gland d. Autoimmune-induced atrophy of the thyroid gland

22. d. In America, atrophy from Graves' disease or Hashimoto's thyroiditis are autoimmune disorders that eventually destroy the thyroid gland, leading to primary hypothyroidism. Worldwide, iodine deficiency is the most common cause. Thyroid tumors most often result in hyperthyroidism. Secondary hypothyroidism occurs as a result of pituitary failure, and iatrogenic hypothyroidism results from thyroidectomy or radiation of the thyroid gland.

25. A patient with hypothyroidism is treated with levothyroxine (Synthroid). What should the nurse include when teaching the patient about this therapy? a. Explain that alternate-day dosage may be used if side effects occur. b. Provide written instruction for all information related to the drug therapy. c. Assure the patient that a return to normal function will occur with replacement therapy. d. Inform the patient that the drug must be taken until the hormone balance is reestablished.

25. b. Because of the mental sluggishness, inattentiveness, and memory loss that occur with hypothyroidism, it is important to provide written instructions and repeat information when teaching the patient. Replacement therapy must be taken for life and alternate-day dosing is not therapeutic. Although most patients return to a normal state with treatment, cardiovascular conditions and psychoses may persist.

27. What is an appropriate nursing intervention for the patient with hyperparathyroidism? a. Pad side rails as a seizure precaution. b. Increase fluid intake to 3000 to 4000 mL daily. c. Maintain bed rest to prevent pathologic fractures. d. Monitor the patient for Trousseau's and Chvostek's signs.

27. b. A high fluid intake is indicated in hyperparathyroidism to dilute the hypercalcemia and flush the kidneys so that calcium stone formation is reduced. Seizures are not associated with hyperparathyroidism. The patient with hyperparathyroidism is at risk for pathologic fractures resulting from decreased bone density, but mobility is encouraged to promote bone calcification. Impending tetany of hypoparathyroidism after parathyroidectomy can be noted with Trousseau's and Chvostek's signs.

29. When the patient with parathyroid disease experiences symptoms of hypocalcemia, what is a measure that can be used to temporarily raise serum calcium levels? a. Administer IV normal saline. c. Administer oral phosphorus supplements. b. Have patient rebreathe in a paper bag. d. Administer furosemide (Lasix) as ordered

29. b. Rebreathing in a paper bag promotes carbon dioxide retention in the blood, which lowers pH and creates an acidosis. An acidemia enhances the solubility and ionization of calcium, increasing the proportion of total body calcium available in physiologically active form and relieving the symptoms of hypocalcemia. Saline promotes calcium excretion, as does furosemide. Phosphate levels in the blood are reciprocal to calcium and an increase in phosphate promotes calcium excretion.

83. The nurse is preparing to administer the following medications. Which medication should the nurse question administering? 1. The thyroid hormone to the client who does not have a T3, T4 level. 2. The regular insulin to the client with a blood glucose level of 210 mg/dL. 3. The loop diuretic to the client with a potassium level of 3.3 mEq/L. 4. The cardiac glycoside to the client who has a digoxin level of 1.4 mg/dL.

3. The loop diuretic to the client with a potassium level of 3.3 mEq/L. / 3. This potassium level is below normal, which is 3.5 to 5.5 mEq/L. Therefore, the nurse should question administering this medication because loop diuretics cause potassium loss in the urine TEST-TAKING HINT: When administering medication, the nurse must know when to question the medication, how to know it is effective, and what must be taught to keep the client safe while taking the medication. The test taker may want to turn the question around and say, "I should give this medication."

4. After thyroid surgery, the nurse suspects damage or removal of the parathyroid glands when the patient develops a. muscle weakness and weight loss. b. hyperthermia and severe tachycardia. c. hypertension and difficulty swallowing. d. laryngospasms and tingling in the hands and feet.

4. Correct answer: d Rationale: Painful tonic spasms of smooth and skeletal muscles can cause laryngospasms that may compromise breathing. These spasms may be related to tetany, which occurs if the parathyroid glands are removed or damaged during surgery, which leads to hypocalcemia.

76. Which nursing intervention should be included in the plan of care for the client diagnosed with hyperthyroidism? 1. Increase the amount of fiber in the diet. 2. Encourage a low-calorie, low-protein diet. 3. Decrease the client's fluid intake to 1,000 mL/day. 4. Provide six (6) small, well-balanced meals a day.

4. Provide six (6) small, well-balanced meals a day. / 4. The client with hyperthyroidism has an increased appetite; therefore, well-balanced meals served several times throughout the day will help with the client's constant hunger. TEST-TAKING HINT: If the test taker knows the metabolism is increased with hyperthyroidism, then increasing the food intake is the most appropriate choice.

499. A client is admitted to an emergency department, and a diagnosis of myxedema coma is made. Which action should the nurse prepare to carry out initially? 1. Warm the client. 2. Maintain a patent airway. 3. Administer thyroid hormone. 4. Administer fluid replacement.

499. Answer: 2 Rationale: Myxedema coma is a rare but serious disorder that results from persistently low thyroid production. Coma can be precipitated by acute illness, rapid withdrawal of thyroid medication, anesthesia and surgery, hypothermia, and the use of sedatives and opioid analgesics. In myxedema coma, the initial nursing action is to maintain a patent airway. Oxygen should be administered, followed by fluid replacement, keeping the client warm, monitoring vital signs, and administering thyroid hormones by the intravenous route. Test-Taking Strategy: Note the strategic word, initially. All the options are appropriate interventions, but use the ABCs—airway, breathing, and circulation—in selecting the correct option.

502. The nurse is completing an assessment on a client who is being admitted for a diagnostic workup for primary hyperparathyroidism. Which client complaints would be characteristic of this disorder? Select all that apply. icon01-9780323358415 1. Polyuria icon01-9780323358415 2. Headache icon01-9780323358415 3. Bone pain icon01-9780323358415 4. Nervousness icon01-9780323358415 5. Weight gain

502. Answer: 1, 3 Rationale: The role of parathyroid hormone (PTH) in the body is to maintain serum calcium homeostasis. In hyperparathyroidism, PTH levels are high, which causes bone resorption (calcium is pulled from the bones). Hypercalcemia occurs with hyperparathyroidism. Elevated serum calcium levels produce osmotic diuresis and thus polyuria. This diuresis leads to dehydration (weight loss rather than weight gain). Loss of calcium from the bones causes bone pain. Options 2, 4, and 5 are not associated with hyperparathyroidism. Some gastrointestinal symptoms include anorexia, nausea, vomiting, and constipation. Test-Taking Strategy: Focus on the subject, assessment findings in hyperparathyroidism. Think about the pathophysiology associated with hyperparathyroidism. Remember that hypercalcemia is associated with this disorder and that hypercalcemia leads to diuresis, and that calcium loss from bone leads to bone pain.

503. The nurse is teaching a client with hyperparathyroidism how to manage the condition at home. Which response by the client indicates the need for additional teaching? 1. "I should consume less than 1 liter of fluid per day." 2. "I should use my treadmill or go for walks daily." 3. "I should follow a moderate-calcium, high-fiber diet." 4. "My alendronate helps keep calcium from coming out of my bones."

503. Answer: 1 Rationale: In hyperparathyroidism, clients experience excess parathyroid hormone (PTH) secretion. A role of PTH in the body is to maintain serum calcium homeostasis. When PTH levels are high, there is excess bone resorption (calcium is pulled from the bones). In clients with elevated serum calcium levels, there is a risk of nephrolithiasis. One to two liters of fluids daily should be encouraged to protect the kidneys and decrease the risk of nephrolithiasis. Moderate physical activity, particularly weight-bearing activity, minimizes bone resorption and helps protect against pathological fracture. Walking, as an exercise, should be encouraged in the client with hyperparathyroidism. Even though serum calcium is already high, clients should follow a moderate-calcium diet, because a low-calcium diet will surge PTH. Calcium causes constipation, so a diet high in fiber is recommended. Alendronate is a bisphosphate that inhibits bone resorption. In bone resorption, bone is broken down and calcium is deposited into the serum. Test-Taking Strategy: Note the strategic words, need for additional teaching. These words indicate a negative event query and the need to select the incorrect statement. Consider the pathophysiology of hyperparathyroidism. Hyperparathyroidism leads to bone demineralization, which places the client at risk for pathological fracture, and high serum calcium, which places the client at risk for nephrolithiasis. Knowing that fluids should be encouraged rather than limited to help prevent nephrolithiasis should direct you to the correct option.

509. The nurse is preparing a client with a new diagnosis of hypothyroidism for discharge. The nurse determines that the client understands discharge instructions if the client states that which signs and symptoms are associated with this diagnosis? Select all that apply. icon01-9780323358415 1. Tremors icon01-9780323358415 2. Weight loss icon01-9780323358415 3. Feeling cold icon01-9780323358415 4. Loss of body hair icon01-9780323358415 5. Persistent lethargy icon01-9780323358415 6. Puffiness of the face

509. Answer: 3, 4, 5, 6 Rationale: Feeling cold, hair loss, lethargy, and facial puffiness are signs of hypothyroidism. Tremors and weight loss are signs of hyperthyroidism. Test-Taking Strategy: Focus on the subject, signs and symptoms associated with hypothyroidism. Options 1 and 2 can be eliminated if you remember that in hypothyroidism there is an undersecretion of thyroid hormone that causes the metabolism to slow down.

510. A client has just been admitted to the nursing unit following thyroidectomy. Which assessment is the priority for this client? 1. Hoarseness 2. Hypocalcemia 3. Audible stridor 4. Edema at the surgical site

510. Answer: 3 Rationale: Thyroidectomy is the removal of the thyroid gland, which is located in the anterior neck. It is very important to monitor airway status, as any swelling to the surgical site could cause respiratory distress. Although all of the options are important for the nurse to monitor, the priority nursing action is to monitor the airway. Test-Taking Strategy: Note the strategic word, priority. Use the ABCs—airway, breathing, and circulation, to assist in directing you to the correct option.

511. A client has been diagnosed with hyperthyroidism. The nurse monitors for which signs and symptoms indicating a complication of this disorder? Select all that apply. icon01-9780323358415 1. Fever icon01-9780323358415 2. Nausea icon01-9780323358415 3. Lethargy icon01-9780323358415 4. Tremors icon01-9780323358415 5. Confusion icon01-9780323358415 6. Bradycardia

511. Answer: 1, 2, 4, 5 Rationale: Thyroid storm is an acute and life-threatening complication that occurs in a client with uncontrollable hyperthyroidism. Signs and symptoms of thyroid storm include elevated temperature (fever), nausea, and tremors. In addition, as the condition progresses, the client becomes confused. The client is restless and anxious and experiences tachycardia. Test-Taking Strategy: Focus on the subject, signs and symptoms indicating a complication of hyperthyroidism. Recall that thyroid storm is a complication of hyperthyroidism. Options 3 and 6 can be eliminated if you remember that thyroid storm is caused by the release of thyroid hormones into the bloodstream, causing uncontrollable hyperthyroidism. Lethargy and bradycardia (think: slow down) are signs of hypothyroidism (slow metabolism).

520. A client with hyperthyroidism has been given methimazole. Which nursing considerations are associated with this medication? Select all that apply. 1. icon01-9780323358415Administer methimazole with food. 2. icon01-9780323358415Place the client on a low-calorie, low-protein diet. 3. icon01-9780323358415Assess the client for unexplained bruising or bleeding. 4. icon01-9780323358415Instruct the client to report side and adverse effects such as sore throat, fever, or headaches. 5. icon01-9780323358415Use special radioactive precautions when handling the client's urine for the first 24 hours following initial administration.

520. Answer: 1, 3, 4 Rationale: Common side effects of methimazole include nausea, vomiting, and diarrhea. To address these side effects, this medication should be taken with food. Because of the increase in metabolism that occurs in hyperthyroidism, the client should consume a high-calorie diet. Antithyroid medications can cause agranulocytosis with leukopenia and thrombocytopenia. Sore throat, fever, headache, or bleeding may indicate agranulocytosis and the primary health care provider should be notified immediately. Methimazole is not radioactive and should not be stopped abruptly, due to the risk of thyroid storm. Test-Taking Strategy: Focus on the subject, nursing considerations for administering methimazole. Focus on the client's diagnosis. Think about the pathophysiology associated with the diagnosis and the medication and the actions and effects of antithyroid medications to assist in answering correctly.

7. Lugol's solution helps block ________ of thyroid hormones in thyroid storm. Which of the following are a common side effect of this medication? A. the removal; tophi B. excretion; swollen lymph nodes C. release/ synthesis; taste changes D. movement; hypocalcemia

C. release/ synthesis; taste changes

5. A physician orders Calcium Gluconate IV as treatment for a patient with hypoparathyroidism. The patient's calcium level is 5 mg/dL. Which of the following finding causes you to question this order? A. The patient is taking Digoxin. B. The patient complains of muscle cramping and numbness in the face. C. The patient is taking Aluminum Carbonate. D. The patient's phosphate level is 7 mg/dL.

Calcium gluconate can increase Digoxin toxicity. Therefore, as the nurse you should clarify the order with the physician and make sure the physician is aware the patient is on Digoxin.

2. All of the following are treatments for myxedema coma EXCEPT? A. Corticosteroids B. IV glucose C. Hypotonic IV solutions D. IV Synthroid

HYPERtonic or normal saline solutions are used to treat myxedema coma due to the present of hyponatremia....not HYPOtonic solutions.

A nurse is caring for a client is undergoing a parathyroidectomy. Which potential risks must the nurse consider when caring for this client following surgery? Bronchial stridor Hypocalcemia Negative nitrogen balance Acoustic neuroma

Hypocalcemia The post-operative parathyroidectomy client is at risk for hypocalcemia, because the parathyroid secretes parathyroid hormone (PTH), which increases the calcium level in the bloodstream by causing calcium release from bone tissue. The nurse can expect that the client who undergoes a parathyroidectomy will have serial calcium lab values drawn so the level can be closely monitored. Signs of hypocalcemia include tingling and twitching in the face and extremities.

1. A patient is recovering from a thyroidectomy. The patient starts to complain of tingling and numbness in the face, toes, and fingers. Which of the following findings below warrants attention? A. Ca+ level: 6 mg/dL B. Na+ level: 145 mg/dL C. K+ level: 3.5 mg/dL D. Phosphate level: 4.3 mg/dL

Patients who've had a thyroidectomy are at risk for HYPOparathyroidism, and the symptoms listed in the question are classic signs of hypocalcemia. A normal calcium level is 8.6 to 10.0 mg/dL. Therefore, due to the patient's signs and symptoms and low calcium level of 6 mg/dL this warrants a nursing intervention.

17. A patient is receiving radioactive iodine treatment for hyperthyroidism. What will you include in your patient education to this patient about this type of treatment? A. Taste changes and swollen salivary glands B. Constipation C. Excessive thirst D. Sun protection

The answer is A: taste changes and swollen salivary glands

1. Which of the following patients are MOST at risk for developing myxedema coma? A. A 28 year old female who is prescribed Methimazole. B. A 75 year old female who is taking Lithium and reports missing several doses of Synthroid. C. A 69 year old male experiencing nausea and vomiting for 4 days. D. A 55 year old male with a history of diabetes and is insulin dependent.

The answer is B. Elderly females are at most risk for myxedema coma. In addition, Lithium decreases thyroid hormone and if the patient has missed several doses of Synthroid (used to treat HYPOthyroidism) this puts them at the greatest risk of myxedema coma.

3. A patient is post-opt from a thyroidectomy for treatment of Grave's Disease. When you walk into the patient's room to perform an assessment, which of the following findings causes the MOST concern and needs nursing intervention? A. The patient complains of a pain rating of 4 on 1-10 at the surgical site. B. The patient is positioned in supine position. C. The patient's Foley catheter is draining 50 cc of urine per hour. D. The patient is splinting the neck while coughing and deep breathing.

The answer is B. The MOST concerning option is that the patient is in the supine position. The patient should be in SEMI-FOWLER'S position after a thyroidectomy to prevent excessive swelling/bleeding and pressure on the site.

2. Which of the following patients are MOST at risk for hypoparathyroidism? A. A 75 year-old female who is diabetic and takes Os-Cal daily. B. A 59 year-old male with a Mg+ level of 0.9 mg/dL. C. A 85 year-old female complaining of flank pain and constipation. D. A 19 year-old male with a Ca+ level of 8.9 mg/dL.

The answer is B. This patient is experiencing HYPOmagnesemia which is a cause of HYPOparathyroidism and is most at risk for developing this condition.

1. Fill in the blank regarding the negative feedback loop for thyroid hormone production: The ______________ produces TRH (Thyrotropin-Releasing Hormone) which causes the anterior pituitary gland to produce _______________ which in turn causes the thyroid gland to release _______ and _______. A. Thalamus, CRH (Corticotropin-releasing hormone) TSH (thyroid-stimulating hormone) and T4 B. Hypothalamus, TSH (thyroid-stimulating hormone), T3 and T4 C. Posterior pituitary gland, TSH (thyroid-stimulating hormone), T3 and T4 D. Hypothalamus, CRH (Corticotropin-releasing hormone), TSH (thyroid-stimulating hormone), T3 and TSH

The answer is B: Hypothalamus, TSH (thyroid-stimulating hormone), T3 and T4

10. A patient was recently discharged home for treatment of hypothyroidism and was ordered to take Synthroid for treatment. The patient is re-admitted with signs and symptoms of the following: heart rate 42, blood pressure 70/56, blood glucose 55, and body temperature of 96.8 'F. The patient is very fatigued and drowsy. The family reports the patient has not been taking Synthroid since being discharged home from the hospital. Which of the following conditions is this patient most likely experiencing? A. Thryoid Storm B. Myxedema Coma C. Iodism D. Toxic Nodular Goiter

The answer is B: Myxedema Coma...The red flags in this question are the patient's signs/symptoms and the report from the family the patient hasn't been taking the prescribed Synthroid. The patient is showing signs and symptoms of extreme hypothyroidism known as Myxedema coma (which is life-threatening if not treated).

1. Which of the following statements are CORRECT about Grave's Disease? A. Grave's Disease is caused by independently functioning nodular goiters producing excessive amounts of T3 and T4. B. Grave's Disease is a complication of untreated hypothyroidism. C. Grave's Disease is caused by an autoimmune condition where the body produces an antibody called TSI (which acts like TSH on the body). D. Grave's Disease patients do not present with protruding eyes or a goiter, as in Toxic Nodular Goiter (TNG)

The answer is C.

5. A patient is being treated for Grave's Disease. They have a health history of type 1 diabetes, breast cancer, eczema, and hypertension. The physician orders Inderal. What important information will you include in their discharge teaching about this medication? A. Importance of taking the medication only as needed for symptoms. B. Avoid aged cheeses and wines while taking this medication. C. Monitor blood glucose levels closely because this medication can mask the signs and symptoms of hypoglycemia. D. Monitor heart rate regularly because this medication will increase the heart rate.

The answer is C. Inderal is a beta-blocker which can cause masking of hypoglycemia in diabetic patients. Therefore, the patient must monitor blood glucose levels closely while taking this medication to treat Grave's Disease.

7. Which of the following are NOT a cause of myxedema coma? A. Illness B. Sedatives C. Iodine Toxicity D. Thyroidectomy

The answer is C. Iodine toxicity can cause HYPERthyroidism issues not HYPOthyroidism issues.

3. A patient is receiving treatment for myxedema coma with IV Synthroid. Which of the following findings would require nursing intervention for this patient? A. Blood glucose 75 B. Sodium level of 138 C. A physician's order for Fentanyl 0.25 mcg every 2 hours for pain D. Temperature 98.9 'F

The answer is C. The other options are normal readings...however, patients with myxedema coma should not receive sedatives or narcotics (Fentanyl is a narcotic) because these patients are very sensitive to them. Therefore, the nurse should intervene and question the doctor's order.

5. A patient taking IV Synthroid starts to complain of feeling hot and chest pain. On assessment, you find that the heart rate is 125 bpm and blood pressure is 200/103. You immediately notify the physician of the patient's condition and receive orders for lab work. Based on the patient's signs and symptoms, what is the MOST important lab result at this time to determine the cause of the patient's symptoms?\ A. Potassium level B. Thyroid levels C. Calcium level D. Sodium level

The answer is C. The patient is receiving IV Synthroid. Therefore, the nurse must monitor for toxicity which would present with elevated THYROID levels. Remember Synthroid is a thyroid replacement medication. This would present with the patient complaining of feeling hot, chest pain, tachycardia, and hypertension (similar signs and symptoms of HYPERthyroidism).

15. ___________ is an autoimmune disorder where the body attacks the thyroid gland that causes it to stop releasing T3 and T4. The patient is likely to have the typical signs/symptoms of hypothyroidism, however, they may present with what other sign as well? A. Myxedema coma; joint pain B. Thyroid storm; memory loss C. Hashimoto's Thyroiditis; goiter D. Toxic nodular goiter (TNG); goiter

The answer is C: Hashimoto's Thyroiditis; goiter

4. A patient is being discharged home for treatment of hypothyroidism. Which medication is most commonly prescribed for this condition? A. Tapazole B. PTU (Propylthiouracil) C. Synthroid D. Inderal

The answer is C: Synthroid is the only medication listed that treats hypothyroidism. All the other medications are used for hyperthyroidism.

6. A patient is recovering from myxedema coma and will be discharged tomorrow. What will you include in their discharge teaching? A. Avoiding green leafy vegetables. B. Importance of taking Tapazole exactly as prescribed at the same time every day. C. Limiting foods with Iodine such as kelp, dairy, and eggs. D. Importance of taking Synthroid in the morning without any food.

The answer is D. Synthroid should be taken in the morning without food so absorption is not affected. All the other options are incorrect discharge education for patients suffering from myxedema coma.

3. A patient has an extremely high T3 and T4 level. Which of the following signs and symptoms DO NOT present with this condition? A. Weight loss B. Intolerance to heat C. Smooth skin D. Hair loss

The answer is D: Hair loss

2. A patient reports they do not eat enough iodine in their diet. What condition are they most susceptible to? A. Pheochromocytoma B. Hyperthyroidism C. Thyroid Storm D. Hypothyroidism

The answer is D: Hypothyroidism...Iodine helps make T3 and T4....if a person does not consume enough iodine they are at risk for developing HYPOTHYROIDISM.

9. Which of the following are treatment options for hyperthyroidism? Please select all that apply: A. Thyroidectomy B. Methimazole C. Liothyronine Sodium "Cytomel" D. Radioactive Iodine

The answers are A, B,and D. Liothyronine Sodium "Cytomel" is a treatment for hypothyroidism. All the other options are for hyperthyroidism.

6. Select all that apply: Which of the following are signs and symptoms of Grave's Disease: A. Heat Intolerance B. Weight gain C. Bradycardia D. Goiter E. Pretibial Myxedema F. Cold intolerance G. Ophthalmopathy changes H. Fast Heart Rate

The answers are A, D, E, G, and H. The only options that are NOT signs and symptoms of Grave's disease are: weight gain, bradycardia, and cold intolerance....these are S & S of HYPOthyroidism.

6. The thyroid hormones, T3 and T4, play many roles in the human body. Which of the following functions are performed by T3 and T4? Note: Select all that apply A. Storing calories B. Increasing the Heart Rate C. Stimulating the Sympathetic Nervous System D. Decreasing the body's temperature E. Regulating TSH produced by the anterior pituitary gland

The answers are B, C, and E. T3 and T4 burn calories (not store them) and increases body temperature (not decrease).

4. A patient is 6 hours post-opt from thyroid surgery. The patient's calcium level is 5 and phosphate level is 4.2. What physical signs and symptoms would NOT present with these findings? (Select-all-that-apply) A. Bronchospasm B. Constipation C. Numbness and tingling in the face D. Positive Chvostek's Sign E. Absent Trousseau's Sign F. Hypertension

The answers are B, E, and F. In this scenario, the patient is at risk for HYPOparathyroidism, especially considering the lab levels. A normal calcium level is 8.9 to 10 mg/dL and phosphate level is 2.7 to 4.5 mg/dL. In HYPOparathyroidism, HYPOcalcemia and HYPERphosphatemia is present (which is the case here). Constipation, absent Trousseau's Sign, and hypertension are not seen in HYPOparathyroidism.

11. This medication is used to treat hyperparathyroidism in patients with chronic renal failure. It works by mimicking the role of calcium in the blood and tricks the parathyroid gland into stop secreting PTH (parathyroid hormone). Which of the following medications does this describe below? A. Calcitonin B. Fosamax C. Lasix D. Sensipar

The description in this question describes the action of Calcimimetics. Sensipar is the only Calcmimetic in this option.

4. A patient taking Tapazole reports feeling dizzy, intolerant to cold, and tired. On assessment, you note the patient's heart rate is 45 and blood pressure is 70/30. What is the most likely cause? A. Antithyroid toxicity B. Agranulocytosis C. Thyroid storm D. Bronchospasm

The patient may be experiencing antithyroid toxicity (too much of the antithyroid medication). This will causes signs and symptoms of hypothyroidism which can lead to a myxedema coma, if not treated immediately.


Conjuntos de estudio relacionados

Measurement of Energy in Food and During Physical Activity

View Set

OB Ch. 11 - Communication, conflict and negotiation

View Set

Identifying child abuse and neglect(DCF,CAAN)

View Set

Mr. Bean - Airplane - Question and Answer

View Set